calculating an integral with green's theorem












0














Let
$ M:{ (x,y) in mathbb{R}_+^{2} : x^2+ 4y^2 leq 4, 1 leq x^2-y^2 } $



I want to use Green's Theorem:



$ I(M)= frac{1}{2} int_{partial M} x_1 dx_2 -x_2 dx_1 $



M is the region where the ellipse and hyperbole are overlapping above the x axis,
so $ int_{partial M }$ must be $ int_1^2 $ right?



now, I am not sure what to use as function I need to integrate.I could not find any similar examples.
Your help is very apreciated :-)










share|cite|improve this question





























    0














    Let
    $ M:{ (x,y) in mathbb{R}_+^{2} : x^2+ 4y^2 leq 4, 1 leq x^2-y^2 } $



    I want to use Green's Theorem:



    $ I(M)= frac{1}{2} int_{partial M} x_1 dx_2 -x_2 dx_1 $



    M is the region where the ellipse and hyperbole are overlapping above the x axis,
    so $ int_{partial M }$ must be $ int_1^2 $ right?



    now, I am not sure what to use as function I need to integrate.I could not find any similar examples.
    Your help is very apreciated :-)










    share|cite|improve this question



























      0












      0








      0







      Let
      $ M:{ (x,y) in mathbb{R}_+^{2} : x^2+ 4y^2 leq 4, 1 leq x^2-y^2 } $



      I want to use Green's Theorem:



      $ I(M)= frac{1}{2} int_{partial M} x_1 dx_2 -x_2 dx_1 $



      M is the region where the ellipse and hyperbole are overlapping above the x axis,
      so $ int_{partial M }$ must be $ int_1^2 $ right?



      now, I am not sure what to use as function I need to integrate.I could not find any similar examples.
      Your help is very apreciated :-)










      share|cite|improve this question















      Let
      $ M:{ (x,y) in mathbb{R}_+^{2} : x^2+ 4y^2 leq 4, 1 leq x^2-y^2 } $



      I want to use Green's Theorem:



      $ I(M)= frac{1}{2} int_{partial M} x_1 dx_2 -x_2 dx_1 $



      M is the region where the ellipse and hyperbole are overlapping above the x axis,
      so $ int_{partial M }$ must be $ int_1^2 $ right?



      now, I am not sure what to use as function I need to integrate.I could not find any similar examples.
      Your help is very apreciated :-)







      real-analysis integration






      share|cite|improve this question















      share|cite|improve this question













      share|cite|improve this question




      share|cite|improve this question








      edited Nov 28 '18 at 19:28

























      asked Nov 27 '18 at 15:17









      wondering1123

      10011




      10011






















          2 Answers
          2






          active

          oldest

          votes


















          2














          I think you mean $displaystyle I( M) =frac{1}{2}int _{partial M} xdy-ydx$



          Taking $displaystyle I( M)$ as written above, by green's theorem this is the area of region M



          enter image description here



          Blue shaded region is the Region M.
          begin{gather*}
          therefore Area of M=I( M) =2int ^{frac{3}{5}}_{-frac{3}{5}}int ^{2sqrt{1-y^{2}}}_{^{sqrt{y^{2} +1}}} dxdy\
          or I( M) =left[ ysqrt{1-y^{2}} -frac{1}{2}sqrt{y^{2} +1} +arcsin( y) -frac{1}{2} arcsinh( y)right]^{y=frac{3}{5}}_{y=-frac{3}{5}}\
          or I( M) =frac{24}{25} -frac{3sqrt{34}}{25} +2arcsinleft(frac{3}{5}right) - arcsinhleft(frac{3}{5}right)\
          or I( M) approx 0.9785
          end{gather*}






          share|cite|improve this answer





















          • thank you for your answere! Since $ (x,y) in mathbb{R^+}^2 $ are the bounds not quite right, right? Where does the Green's Theorem flows in ?
            – wondering1123
            Nov 27 '18 at 20:54










          • @wondering1123 The formula for area,i.e.,I(M) has been derived from Green's theorem
            – Dikshit Gautam
            Nov 28 '18 at 4:53










          • I am sorry, but thats the way i would normally integrate it..I thought about it and I still can not explain it with the formular $I(M)$ above. would you mind explaining it?
            – wondering1123
            Nov 28 '18 at 19:27










          • You haven't used Green's theorem to evaluate the area. If you use Green's theorem, you will evaluate a line integral, not a double integral.
            – saulspatz
            Nov 28 '18 at 19:38



















          1














          Green's theorem says that $$int_{partial S}{Pdx+Qdy}=intint_S(Q_y-P_x)dxdy$$ with suitable hypotheses on $P,Q,S$. So if we set $P(x,y)=0, Q(x,y)=y,$ we see that the area of $S$ is $$int_{partial S}{y dy}$$ You simply have to parametrize the curves bounding $S$ and integrate the expression for $ydy$.



          $S$ breaks into two congruent regions, so it's enough to compute the area of one of them.enter image description here



          The region on the right is bounded below by a segment the x-axis. If we parameterize $y$ on this axis, we will have $y=0$ of course, so the line integral will be $0$ and we can ignore it. The blue boundary curve runs from $(2,0)$ to the red dot at $(sqrt{1.6},sqrt{.4})$ and we can parameterize along this curve as $y$ as $$
          y=sqrt{{4-x^2over4}}implies dy= {-xdxover 2sqrt{4-x^2}},$$
          so you have to integrate ${-xover4}dx$ from $x=2$ to x=$sqrt{1.6}.$ The direction of the integral is determined because in Green's theorem, the contour of the line integral is oriented counter-clockwise.



          We have $ydy={-xover8}dx,$ so we compute $$int_2^{sqrt{.6}}{-xover8}dx={-x^2over16}biggrrvert_2^{sqrt{.6}}={3.4over16}$$



          Do the same thing for the arc of the hyperbola that bounds the region on the right.






          share|cite|improve this answer























          • thanks for the detailled explanation! It is still quite hard for me to get it..and I need to integrate only for positive $x,y$
            – wondering1123
            Nov 28 '18 at 21:27










          • I'll add a few more details. Wait a bit.
            – saulspatz
            Nov 28 '18 at 21:48










          • I added some details. Note that the contour satisfies $xge0, yge0$ so we are considering only positive $x$ and $y$. What makes you think we aren't?
            – saulspatz
            Nov 28 '18 at 22:01











          Your Answer





          StackExchange.ifUsing("editor", function () {
          return StackExchange.using("mathjaxEditing", function () {
          StackExchange.MarkdownEditor.creationCallbacks.add(function (editor, postfix) {
          StackExchange.mathjaxEditing.prepareWmdForMathJax(editor, postfix, [["$", "$"], ["\\(","\\)"]]);
          });
          });
          }, "mathjax-editing");

          StackExchange.ready(function() {
          var channelOptions = {
          tags: "".split(" "),
          id: "69"
          };
          initTagRenderer("".split(" "), "".split(" "), channelOptions);

          StackExchange.using("externalEditor", function() {
          // Have to fire editor after snippets, if snippets enabled
          if (StackExchange.settings.snippets.snippetsEnabled) {
          StackExchange.using("snippets", function() {
          createEditor();
          });
          }
          else {
          createEditor();
          }
          });

          function createEditor() {
          StackExchange.prepareEditor({
          heartbeatType: 'answer',
          autoActivateHeartbeat: false,
          convertImagesToLinks: true,
          noModals: true,
          showLowRepImageUploadWarning: true,
          reputationToPostImages: 10,
          bindNavPrevention: true,
          postfix: "",
          imageUploader: {
          brandingHtml: "Powered by u003ca class="icon-imgur-white" href="https://imgur.com/"u003eu003c/au003e",
          contentPolicyHtml: "User contributions licensed under u003ca href="https://creativecommons.org/licenses/by-sa/3.0/"u003ecc by-sa 3.0 with attribution requiredu003c/au003e u003ca href="https://stackoverflow.com/legal/content-policy"u003e(content policy)u003c/au003e",
          allowUrls: true
          },
          noCode: true, onDemand: true,
          discardSelector: ".discard-answer"
          ,immediatelyShowMarkdownHelp:true
          });


          }
          });














          draft saved

          draft discarded


















          StackExchange.ready(
          function () {
          StackExchange.openid.initPostLogin('.new-post-login', 'https%3a%2f%2fmath.stackexchange.com%2fquestions%2f3015889%2fcalculating-an-integral-with-greens-theorem%23new-answer', 'question_page');
          }
          );

          Post as a guest















          Required, but never shown

























          2 Answers
          2






          active

          oldest

          votes








          2 Answers
          2






          active

          oldest

          votes









          active

          oldest

          votes






          active

          oldest

          votes









          2














          I think you mean $displaystyle I( M) =frac{1}{2}int _{partial M} xdy-ydx$



          Taking $displaystyle I( M)$ as written above, by green's theorem this is the area of region M



          enter image description here



          Blue shaded region is the Region M.
          begin{gather*}
          therefore Area of M=I( M) =2int ^{frac{3}{5}}_{-frac{3}{5}}int ^{2sqrt{1-y^{2}}}_{^{sqrt{y^{2} +1}}} dxdy\
          or I( M) =left[ ysqrt{1-y^{2}} -frac{1}{2}sqrt{y^{2} +1} +arcsin( y) -frac{1}{2} arcsinh( y)right]^{y=frac{3}{5}}_{y=-frac{3}{5}}\
          or I( M) =frac{24}{25} -frac{3sqrt{34}}{25} +2arcsinleft(frac{3}{5}right) - arcsinhleft(frac{3}{5}right)\
          or I( M) approx 0.9785
          end{gather*}






          share|cite|improve this answer





















          • thank you for your answere! Since $ (x,y) in mathbb{R^+}^2 $ are the bounds not quite right, right? Where does the Green's Theorem flows in ?
            – wondering1123
            Nov 27 '18 at 20:54










          • @wondering1123 The formula for area,i.e.,I(M) has been derived from Green's theorem
            – Dikshit Gautam
            Nov 28 '18 at 4:53










          • I am sorry, but thats the way i would normally integrate it..I thought about it and I still can not explain it with the formular $I(M)$ above. would you mind explaining it?
            – wondering1123
            Nov 28 '18 at 19:27










          • You haven't used Green's theorem to evaluate the area. If you use Green's theorem, you will evaluate a line integral, not a double integral.
            – saulspatz
            Nov 28 '18 at 19:38
















          2














          I think you mean $displaystyle I( M) =frac{1}{2}int _{partial M} xdy-ydx$



          Taking $displaystyle I( M)$ as written above, by green's theorem this is the area of region M



          enter image description here



          Blue shaded region is the Region M.
          begin{gather*}
          therefore Area of M=I( M) =2int ^{frac{3}{5}}_{-frac{3}{5}}int ^{2sqrt{1-y^{2}}}_{^{sqrt{y^{2} +1}}} dxdy\
          or I( M) =left[ ysqrt{1-y^{2}} -frac{1}{2}sqrt{y^{2} +1} +arcsin( y) -frac{1}{2} arcsinh( y)right]^{y=frac{3}{5}}_{y=-frac{3}{5}}\
          or I( M) =frac{24}{25} -frac{3sqrt{34}}{25} +2arcsinleft(frac{3}{5}right) - arcsinhleft(frac{3}{5}right)\
          or I( M) approx 0.9785
          end{gather*}






          share|cite|improve this answer





















          • thank you for your answere! Since $ (x,y) in mathbb{R^+}^2 $ are the bounds not quite right, right? Where does the Green's Theorem flows in ?
            – wondering1123
            Nov 27 '18 at 20:54










          • @wondering1123 The formula for area,i.e.,I(M) has been derived from Green's theorem
            – Dikshit Gautam
            Nov 28 '18 at 4:53










          • I am sorry, but thats the way i would normally integrate it..I thought about it and I still can not explain it with the formular $I(M)$ above. would you mind explaining it?
            – wondering1123
            Nov 28 '18 at 19:27










          • You haven't used Green's theorem to evaluate the area. If you use Green's theorem, you will evaluate a line integral, not a double integral.
            – saulspatz
            Nov 28 '18 at 19:38














          2












          2








          2






          I think you mean $displaystyle I( M) =frac{1}{2}int _{partial M} xdy-ydx$



          Taking $displaystyle I( M)$ as written above, by green's theorem this is the area of region M



          enter image description here



          Blue shaded region is the Region M.
          begin{gather*}
          therefore Area of M=I( M) =2int ^{frac{3}{5}}_{-frac{3}{5}}int ^{2sqrt{1-y^{2}}}_{^{sqrt{y^{2} +1}}} dxdy\
          or I( M) =left[ ysqrt{1-y^{2}} -frac{1}{2}sqrt{y^{2} +1} +arcsin( y) -frac{1}{2} arcsinh( y)right]^{y=frac{3}{5}}_{y=-frac{3}{5}}\
          or I( M) =frac{24}{25} -frac{3sqrt{34}}{25} +2arcsinleft(frac{3}{5}right) - arcsinhleft(frac{3}{5}right)\
          or I( M) approx 0.9785
          end{gather*}






          share|cite|improve this answer












          I think you mean $displaystyle I( M) =frac{1}{2}int _{partial M} xdy-ydx$



          Taking $displaystyle I( M)$ as written above, by green's theorem this is the area of region M



          enter image description here



          Blue shaded region is the Region M.
          begin{gather*}
          therefore Area of M=I( M) =2int ^{frac{3}{5}}_{-frac{3}{5}}int ^{2sqrt{1-y^{2}}}_{^{sqrt{y^{2} +1}}} dxdy\
          or I( M) =left[ ysqrt{1-y^{2}} -frac{1}{2}sqrt{y^{2} +1} +arcsin( y) -frac{1}{2} arcsinh( y)right]^{y=frac{3}{5}}_{y=-frac{3}{5}}\
          or I( M) =frac{24}{25} -frac{3sqrt{34}}{25} +2arcsinleft(frac{3}{5}right) - arcsinhleft(frac{3}{5}right)\
          or I( M) approx 0.9785
          end{gather*}







          share|cite|improve this answer












          share|cite|improve this answer



          share|cite|improve this answer










          answered Nov 27 '18 at 16:14









          Dikshit Gautam

          795




          795












          • thank you for your answere! Since $ (x,y) in mathbb{R^+}^2 $ are the bounds not quite right, right? Where does the Green's Theorem flows in ?
            – wondering1123
            Nov 27 '18 at 20:54










          • @wondering1123 The formula for area,i.e.,I(M) has been derived from Green's theorem
            – Dikshit Gautam
            Nov 28 '18 at 4:53










          • I am sorry, but thats the way i would normally integrate it..I thought about it and I still can not explain it with the formular $I(M)$ above. would you mind explaining it?
            – wondering1123
            Nov 28 '18 at 19:27










          • You haven't used Green's theorem to evaluate the area. If you use Green's theorem, you will evaluate a line integral, not a double integral.
            – saulspatz
            Nov 28 '18 at 19:38


















          • thank you for your answere! Since $ (x,y) in mathbb{R^+}^2 $ are the bounds not quite right, right? Where does the Green's Theorem flows in ?
            – wondering1123
            Nov 27 '18 at 20:54










          • @wondering1123 The formula for area,i.e.,I(M) has been derived from Green's theorem
            – Dikshit Gautam
            Nov 28 '18 at 4:53










          • I am sorry, but thats the way i would normally integrate it..I thought about it and I still can not explain it with the formular $I(M)$ above. would you mind explaining it?
            – wondering1123
            Nov 28 '18 at 19:27










          • You haven't used Green's theorem to evaluate the area. If you use Green's theorem, you will evaluate a line integral, not a double integral.
            – saulspatz
            Nov 28 '18 at 19:38
















          thank you for your answere! Since $ (x,y) in mathbb{R^+}^2 $ are the bounds not quite right, right? Where does the Green's Theorem flows in ?
          – wondering1123
          Nov 27 '18 at 20:54




          thank you for your answere! Since $ (x,y) in mathbb{R^+}^2 $ are the bounds not quite right, right? Where does the Green's Theorem flows in ?
          – wondering1123
          Nov 27 '18 at 20:54












          @wondering1123 The formula for area,i.e.,I(M) has been derived from Green's theorem
          – Dikshit Gautam
          Nov 28 '18 at 4:53




          @wondering1123 The formula for area,i.e.,I(M) has been derived from Green's theorem
          – Dikshit Gautam
          Nov 28 '18 at 4:53












          I am sorry, but thats the way i would normally integrate it..I thought about it and I still can not explain it with the formular $I(M)$ above. would you mind explaining it?
          – wondering1123
          Nov 28 '18 at 19:27




          I am sorry, but thats the way i would normally integrate it..I thought about it and I still can not explain it with the formular $I(M)$ above. would you mind explaining it?
          – wondering1123
          Nov 28 '18 at 19:27












          You haven't used Green's theorem to evaluate the area. If you use Green's theorem, you will evaluate a line integral, not a double integral.
          – saulspatz
          Nov 28 '18 at 19:38




          You haven't used Green's theorem to evaluate the area. If you use Green's theorem, you will evaluate a line integral, not a double integral.
          – saulspatz
          Nov 28 '18 at 19:38











          1














          Green's theorem says that $$int_{partial S}{Pdx+Qdy}=intint_S(Q_y-P_x)dxdy$$ with suitable hypotheses on $P,Q,S$. So if we set $P(x,y)=0, Q(x,y)=y,$ we see that the area of $S$ is $$int_{partial S}{y dy}$$ You simply have to parametrize the curves bounding $S$ and integrate the expression for $ydy$.



          $S$ breaks into two congruent regions, so it's enough to compute the area of one of them.enter image description here



          The region on the right is bounded below by a segment the x-axis. If we parameterize $y$ on this axis, we will have $y=0$ of course, so the line integral will be $0$ and we can ignore it. The blue boundary curve runs from $(2,0)$ to the red dot at $(sqrt{1.6},sqrt{.4})$ and we can parameterize along this curve as $y$ as $$
          y=sqrt{{4-x^2over4}}implies dy= {-xdxover 2sqrt{4-x^2}},$$
          so you have to integrate ${-xover4}dx$ from $x=2$ to x=$sqrt{1.6}.$ The direction of the integral is determined because in Green's theorem, the contour of the line integral is oriented counter-clockwise.



          We have $ydy={-xover8}dx,$ so we compute $$int_2^{sqrt{.6}}{-xover8}dx={-x^2over16}biggrrvert_2^{sqrt{.6}}={3.4over16}$$



          Do the same thing for the arc of the hyperbola that bounds the region on the right.






          share|cite|improve this answer























          • thanks for the detailled explanation! It is still quite hard for me to get it..and I need to integrate only for positive $x,y$
            – wondering1123
            Nov 28 '18 at 21:27










          • I'll add a few more details. Wait a bit.
            – saulspatz
            Nov 28 '18 at 21:48










          • I added some details. Note that the contour satisfies $xge0, yge0$ so we are considering only positive $x$ and $y$. What makes you think we aren't?
            – saulspatz
            Nov 28 '18 at 22:01
















          1














          Green's theorem says that $$int_{partial S}{Pdx+Qdy}=intint_S(Q_y-P_x)dxdy$$ with suitable hypotheses on $P,Q,S$. So if we set $P(x,y)=0, Q(x,y)=y,$ we see that the area of $S$ is $$int_{partial S}{y dy}$$ You simply have to parametrize the curves bounding $S$ and integrate the expression for $ydy$.



          $S$ breaks into two congruent regions, so it's enough to compute the area of one of them.enter image description here



          The region on the right is bounded below by a segment the x-axis. If we parameterize $y$ on this axis, we will have $y=0$ of course, so the line integral will be $0$ and we can ignore it. The blue boundary curve runs from $(2,0)$ to the red dot at $(sqrt{1.6},sqrt{.4})$ and we can parameterize along this curve as $y$ as $$
          y=sqrt{{4-x^2over4}}implies dy= {-xdxover 2sqrt{4-x^2}},$$
          so you have to integrate ${-xover4}dx$ from $x=2$ to x=$sqrt{1.6}.$ The direction of the integral is determined because in Green's theorem, the contour of the line integral is oriented counter-clockwise.



          We have $ydy={-xover8}dx,$ so we compute $$int_2^{sqrt{.6}}{-xover8}dx={-x^2over16}biggrrvert_2^{sqrt{.6}}={3.4over16}$$



          Do the same thing for the arc of the hyperbola that bounds the region on the right.






          share|cite|improve this answer























          • thanks for the detailled explanation! It is still quite hard for me to get it..and I need to integrate only for positive $x,y$
            – wondering1123
            Nov 28 '18 at 21:27










          • I'll add a few more details. Wait a bit.
            – saulspatz
            Nov 28 '18 at 21:48










          • I added some details. Note that the contour satisfies $xge0, yge0$ so we are considering only positive $x$ and $y$. What makes you think we aren't?
            – saulspatz
            Nov 28 '18 at 22:01














          1












          1








          1






          Green's theorem says that $$int_{partial S}{Pdx+Qdy}=intint_S(Q_y-P_x)dxdy$$ with suitable hypotheses on $P,Q,S$. So if we set $P(x,y)=0, Q(x,y)=y,$ we see that the area of $S$ is $$int_{partial S}{y dy}$$ You simply have to parametrize the curves bounding $S$ and integrate the expression for $ydy$.



          $S$ breaks into two congruent regions, so it's enough to compute the area of one of them.enter image description here



          The region on the right is bounded below by a segment the x-axis. If we parameterize $y$ on this axis, we will have $y=0$ of course, so the line integral will be $0$ and we can ignore it. The blue boundary curve runs from $(2,0)$ to the red dot at $(sqrt{1.6},sqrt{.4})$ and we can parameterize along this curve as $y$ as $$
          y=sqrt{{4-x^2over4}}implies dy= {-xdxover 2sqrt{4-x^2}},$$
          so you have to integrate ${-xover4}dx$ from $x=2$ to x=$sqrt{1.6}.$ The direction of the integral is determined because in Green's theorem, the contour of the line integral is oriented counter-clockwise.



          We have $ydy={-xover8}dx,$ so we compute $$int_2^{sqrt{.6}}{-xover8}dx={-x^2over16}biggrrvert_2^{sqrt{.6}}={3.4over16}$$



          Do the same thing for the arc of the hyperbola that bounds the region on the right.






          share|cite|improve this answer














          Green's theorem says that $$int_{partial S}{Pdx+Qdy}=intint_S(Q_y-P_x)dxdy$$ with suitable hypotheses on $P,Q,S$. So if we set $P(x,y)=0, Q(x,y)=y,$ we see that the area of $S$ is $$int_{partial S}{y dy}$$ You simply have to parametrize the curves bounding $S$ and integrate the expression for $ydy$.



          $S$ breaks into two congruent regions, so it's enough to compute the area of one of them.enter image description here



          The region on the right is bounded below by a segment the x-axis. If we parameterize $y$ on this axis, we will have $y=0$ of course, so the line integral will be $0$ and we can ignore it. The blue boundary curve runs from $(2,0)$ to the red dot at $(sqrt{1.6},sqrt{.4})$ and we can parameterize along this curve as $y$ as $$
          y=sqrt{{4-x^2over4}}implies dy= {-xdxover 2sqrt{4-x^2}},$$
          so you have to integrate ${-xover4}dx$ from $x=2$ to x=$sqrt{1.6}.$ The direction of the integral is determined because in Green's theorem, the contour of the line integral is oriented counter-clockwise.



          We have $ydy={-xover8}dx,$ so we compute $$int_2^{sqrt{.6}}{-xover8}dx={-x^2over16}biggrrvert_2^{sqrt{.6}}={3.4over16}$$



          Do the same thing for the arc of the hyperbola that bounds the region on the right.







          share|cite|improve this answer














          share|cite|improve this answer



          share|cite|improve this answer








          edited Nov 28 '18 at 21:58

























          answered Nov 28 '18 at 20:10









          saulspatz

          14k21329




          14k21329












          • thanks for the detailled explanation! It is still quite hard for me to get it..and I need to integrate only for positive $x,y$
            – wondering1123
            Nov 28 '18 at 21:27










          • I'll add a few more details. Wait a bit.
            – saulspatz
            Nov 28 '18 at 21:48










          • I added some details. Note that the contour satisfies $xge0, yge0$ so we are considering only positive $x$ and $y$. What makes you think we aren't?
            – saulspatz
            Nov 28 '18 at 22:01


















          • thanks for the detailled explanation! It is still quite hard for me to get it..and I need to integrate only for positive $x,y$
            – wondering1123
            Nov 28 '18 at 21:27










          • I'll add a few more details. Wait a bit.
            – saulspatz
            Nov 28 '18 at 21:48










          • I added some details. Note that the contour satisfies $xge0, yge0$ so we are considering only positive $x$ and $y$. What makes you think we aren't?
            – saulspatz
            Nov 28 '18 at 22:01
















          thanks for the detailled explanation! It is still quite hard for me to get it..and I need to integrate only for positive $x,y$
          – wondering1123
          Nov 28 '18 at 21:27




          thanks for the detailled explanation! It is still quite hard for me to get it..and I need to integrate only for positive $x,y$
          – wondering1123
          Nov 28 '18 at 21:27












          I'll add a few more details. Wait a bit.
          – saulspatz
          Nov 28 '18 at 21:48




          I'll add a few more details. Wait a bit.
          – saulspatz
          Nov 28 '18 at 21:48












          I added some details. Note that the contour satisfies $xge0, yge0$ so we are considering only positive $x$ and $y$. What makes you think we aren't?
          – saulspatz
          Nov 28 '18 at 22:01




          I added some details. Note that the contour satisfies $xge0, yge0$ so we are considering only positive $x$ and $y$. What makes you think we aren't?
          – saulspatz
          Nov 28 '18 at 22:01


















          draft saved

          draft discarded




















































          Thanks for contributing an answer to Mathematics Stack Exchange!


          • Please be sure to answer the question. Provide details and share your research!

          But avoid



          • Asking for help, clarification, or responding to other answers.

          • Making statements based on opinion; back them up with references or personal experience.


          Use MathJax to format equations. MathJax reference.


          To learn more, see our tips on writing great answers.





          Some of your past answers have not been well-received, and you're in danger of being blocked from answering.


          Please pay close attention to the following guidance:


          • Please be sure to answer the question. Provide details and share your research!

          But avoid



          • Asking for help, clarification, or responding to other answers.

          • Making statements based on opinion; back them up with references or personal experience.


          To learn more, see our tips on writing great answers.




          draft saved


          draft discarded














          StackExchange.ready(
          function () {
          StackExchange.openid.initPostLogin('.new-post-login', 'https%3a%2f%2fmath.stackexchange.com%2fquestions%2f3015889%2fcalculating-an-integral-with-greens-theorem%23new-answer', 'question_page');
          }
          );

          Post as a guest















          Required, but never shown





















































          Required, but never shown














          Required, but never shown












          Required, but never shown







          Required, but never shown

































          Required, but never shown














          Required, but never shown












          Required, but never shown







          Required, but never shown







          Popular posts from this blog

          Quarter-circle Tiles

          build a pushdown automaton that recognizes the reverse language of a given pushdown automaton?

          Mont Emei